2023 AIME II Problems/Problem 12
Revision as of 18:05, 16 February 2023 by Stevenyiweichen (talk | contribs) (Created page with "==Solution== Because <math>M</math> is the midpoint of <math>BC</math>, following from the Steward's theorem, <math>AM = 2 \sqrt{37}</math>. Because <math>A</math>, <math>B<...")
Solution
Because is the midpoint of , following from the Steward's theorem, .
Because , , , are concyclic, , .
Denote .
In , following from the law of sines,
Thus,
In , following from the law of sines,
Thus,
Taking , we get
In , following from the law of sines,
Thus, Equations (2) and (3) imply
Next, we compute and .
We have
We have
Taking (5) and (6) into (4), we get . Therefore, the answer is .
~Steven Chen (Professor Chen Education Palace, www.professorchenedu.com)